Why E and not A?
Hi I choose E because it seemed the closest but I really had a hard time eliminating A. Could yo...
farnoushsalimian on November 29, 2019
  • June 2010 LSAT
  • SEC3
  • Q8
6
Replies
Why A?
I understand that it cannot be other answer choices, but to me it seems like it cannot be A eithe...
vijetakanabar90 on November 19, 2019
  • June 2010 LSAT
  • SEC3
  • Q15
2
Replies
Could you please explain why C is the correct a...
Could you please explain how C is the correct answer? I know the answer choice is based on simila...
Deidra-McCall on November 4, 2019
  • June 2010 LSAT
  • SEC3
  • Q19
6
Replies
Option D
Hey, I understand why E is correct but can you explain why D is incorrect? Thanks!
Minerva on July 24, 2019
  • June 2010 LSAT
  • SEC3
  • Q5
2
Replies
Why not C?
It seems that if most drivers don't follow the speed limit, the change must be due to another fac...
lizaclark95 on May 1, 2019
  • June 2010 LSAT
  • SEC3
  • Q20
2
Replies
Why is this
Answer D and not B?
Jessica-Killeen on January 31, 2019
  • June 2010 LSAT
  • SEC3
  • Q20
6
Replies
Why B?
I need some help understanding this question...
Joseph on February 5, 2016
  • June 2010 LSAT
  • SEC3
  • Q23
2
Replies
Assumption help
I am having difficulty understanding why E is the correct answer. Please assist!
MJA7 on February 5, 2015
  • June 2010 LSAT
  • SEC3
  • Q24
6
Replies
Question
Why is the answer not B?
Eunbeezy on January 15, 2015
  • June 2010 LSAT
  • SEC3
  • Q22
2
Replies
Please Explain
I do not understand the correct answer
KDA86 on December 10, 2014
  • June 2010 LSAT
  • SEC3
  • Q9
2
Replies